Entry Test - 2 (JEE Eklavya 2023-Earlybird)

You might also like

Download as pdf or txt
Download as pdf or txt
You are on page 1of 45

Entry Test - 2 ( JEE Eklavya 2023-Earlybird )

Questions

1. Choose the correct statement about X-rays


(A)  A photon of cut off wavelength is most energetic.
(B)  A K α photon is more energetic than a K β photon.
(C)  A photon of cut off wavelength is least energetic.
(D)  As we move down the periodic table the energy of ′ K α ′ X-ray photons goes on decreasing.

2. Calculate the inductance of a unit length of a double tape line as shown in figure. The tapes are separated by a
distance h which is considerably less than their width b. Consider tapes to be of infinite length. The tapes carry equal
current in opposite directions, as depicted in the figure.

μ 0h
(a)  
b
μ 0h
(b)  
2b
2μ 0h
(c)  
b
√2μ 0h
(d)  
b

3. A point charge q is placed at a distance r from the centre O of a uncharged spherical shell of inner radius R and outer
radius 2R. The distance r < R. The electric potential at the centre of the shell will be:

(a)   q
4πε 0
q
( )
1
r

1
2R

(b)   4πε r
0

(c)   q
4πε 0 ( )
1
r
+
1
2R

(d)  None of the above


4. In YDSE experiment if a slab whose refractive index (μ) can be varied is placed in front of one of the slits then the
variation of resultant intensity at mid-point of screen with ‘μ’ will be best represented by [assume slits of equal width
and there is no absorption by slab]:

(a)  

(b)  

(c)  

(d)  

5. A transparent material of refractive index μ = √3 is placed in vacuum. The cross-

section EDABC is shown. A narrow beam of light is parallel to the base ADE and is at a height
√3
h = 2 R from it. The curve part AB is quarter circle of radius R. The distance from point A

where the beam meet the base line is :

(a)  ∞
(b)  2R
√3
(c)  
2 (√3 + 1 ) R
3
(d)   R
2
6. Two large parallel plates carry charge of equal magnitude, one positive and the other negative, that is distributed
uniformly over their inner surfaces. Rank the points 1 through 5 according to the magnitude of the electric field at the
points, least to greatest.

(a)  1, 2, 3, 4, 5
(b)  5, 4, 3, 2, 1
(c)  1, 4 and 5 tie, then 2 and 3 tie
(d)  2 and 3 tie, then 1 and 4 tie, then 5

7. A thin flexible wire of length L is connected to two adjacent fixed points and carries a current I in the clockwise
direction, as shown in the figure. When the system is put in a uniform magnetic field of strength B going into the plane
of the paper, the wire takes the shape of a circle. The tension in the wire is

(a)  IBL
IBL
(b)  
π
IBL
(c)  

IBL
(d)  

8. The de Broglie wavelength of electron in the 3rd orbit of H-atom is x Å. Find the de Broglie wavelength of electron in the
5th orbit of H-atom
3x
(a)  λ = 5
5x
(b)  λ =
3
2x
(c)  λ =
5
x
(d)  λ =
5

9. A square loop of side ‘2a’, and carrying current ‘2I’, is kept in XZ plane with its centre at origin and sides parallel to ‘X’
and ‘Z’ axes. A long wire carrying current ‘I’ is placed parallel to the z-axis and passing through the point (0, b, 0), (b > >
a). Select the INCORRECT statement from among the given below options:
(a)  The coefficient of mutual inductance of this arrangement is zero.
4μ 0I 2a 2
(b)  The magnitude of the torque on the loop about z-axis is given by
πb
2μ 0I 2a 2
(c)  The magnitude of the torque on the loop about z-axis is given by
πb
(d)  The net force on square loop is non-zero.

10. When an LED is in operation, the state of biasing is -


(a)  forward biased
(b)  reversed biased
(c)  can be any of above two
(d)  depends on usage of photodiode

11. An electrical power line, having a total resistance of 5 ohms delivers 5 kW at ‘V’ volts. The efficiency of the
transmission line is 40%. The value of ‘V’ is approximately:
(a)  65
(b)  258
(c)  129
(d)  220
12. An isolated, uniform sphere of mass ‘M’ and radius ‘R’, has a uniform distribution of charge ‘Q’ in its volume. The
sphere lies in free space. A small element of volume ‘dV’ gently detaches from the surface of the sphere. The left-over
object can still be treated as sphere. Given KQ2 = GM2 . Choose the correct statement from the given below options.
(a)  positive work needs to be done by an external agent in slowly shifting the element to infinity.
After the element gets detached, the sphere and the element push each other until the separation between them
(b)  
becomes infinite.
The element can be converted into a satellite of sphere by providing appropriate velocity in appropriate direction
(c)  
to it.
(d)  The force of interaction between the element and the sphere is zero.

13. A galvanometer of resistance G is converted into a voltmeter of range 0 – 1V by connecting a resistance R1 in series
with it. It is desired to decrease the range of the voltmeter to 0 – 0.2V by connecting an additional resistance R2 in
parallel with R1. Choose the INCORRECT option from the given below options.
(a)  R1 must be greater than 4G to obtain the desired range of 0-0.2V in the described manner.
(b)  R2 = R1(R1 -4G)/4(R1+G)
(c)  R2 = R1(R1 -4G)/(R1+G)
(d)  The overall resistance of the voltmeter will decrease after connecting R2 in the desired manner.

14. An electron is constrained to move along the y-axis with a speed of 0.1 c (c is the speed of light) in the presence of

( )
electromagnetic wave, whose magnetic field is B = 10 − 7sin 1.5 × 10 7t − 5 × 10 − 2x ĵ T. Which of the following options
in INCORRECT ? (given c = 3 × 108 ms–1 and electron charge = 1.6 × 10–19C)
(a)  The maximum electric force experienced by the electron will be 4.8 × 10–18 N
(b)  The magnetic force experienced by the electron will be along z – axis.
(c)  The electric force experienced by the electron will be along z – axis
(d)  Power delivered to the electron by the electric force will be zero.

15. In a small inductor of inductance 10mH , the current becomes 0 from 0.5 mA in 0.02 sec. What is the emf induced ?
(a)  0.5 mV
(b)  0.1 mV
(c)  0.4 mV
(d)  0.25 mV

16. A charged particle of mass ′m′ and charge ′q′ is moving under the influence of uniform electric field E = E 0 î and a
uniform magnetic field B = − B 0k̂ follows a trajectory from P to Q as shown in figure. The velocities at P and Q are
respectively v0 î and − 2v0 ĵ . Then which of the following statements (1,2,3,4) are correct? (Trajectory shown is
schematic and not to scale)
Here E0 , B0 and v0 are all positive constants.

 Statement1: q must be positive


 Statement2: Work done by magnetic field must be negative
 Statement 3: Work done by electric field must be positive
Statement4: q must be negative
(a)  Statement 1,3 are correct
(b)  Statement 3,4 are correct
(c)  Statement 2,3,4 are correct
(d)  Statement 1,2,3 are correct
17. Two plane electromagnetic waves are moving in vacuum in whose Magnetic field vectors are given by

B 1 = B 0cos(ky − ωt) î and B 2 = B 0cos(kx − ωt) k̂. At t = 0, a charge q is at origin with velocity v = 0.5c ĵ (𝑐 is speed of light
in vacuum). The instantaneous force on this charge (all data are in SI units)

( )
(a)  qcB 0 0.5 î + ĵ + 0.5k̂

(b)  qcB (0.5 î + ĵ + k̂ )
0

(c)  qcB ( î + ĵ + 0.5k̂ )
0

(d)  qcB ( î + ĵ + k̂ )
0

18. In the given circuit diagram, a wire is joined between points B & C. Find the current in this wire.

(a)  4/3 ampere from B to C


(b)  4/3 ampere from C to B
(c)  2/3 ampere from B to C
(d)  2/3 ampere from C to B

19. A vessel of depth 2ℎ is filled with a liquid of refractive index μ1 = 3 in upper one-third of the vessel and with a liquid of
refractive index μ2 = 1.5 in bottom two third. The liquids are immiscible. The apparent depth of inner surface of the
bottom of the vessel, when viewed from directly above the liquid surface, will be
10h
(a)  
9
10h
(b)   7
10h
(c)  
3
10h
(d)  
11

20. If the percentage decrease in the de Broglie wavelength of a particle is 60% then what is the ratio of the final to initial
kinetic energy of the particle.
(a)  5.25
(b)  6.25
(c)  4.25
(d)  3.25

21. Radiation with wavelength 


λ  falls on a metal surface to produce photoelectrons. The electrons are made to enter a
uniform magnetic field of 2 × 10-4 T. If the radius of largest circular path followed by electron is 20 mm, and the work
function of metal is 2 eV, then find λ   . Take mass of electron to be = 10-30 kg
(a)  3780 Å
(b)  3880 Å
(c)  3980 Å
(d)  4080 Å
22. →
Consider a small electric dipole placed at origin, having dipole moment p. Consider a point in space with position
vector →r, such that p.
→ →
r = 0. Then the electric field due to dipole at this point is:
(a)  Zero
(b)  Parallel to dipole moment
(c)  Anti parallel to dipole moment
(d)  Data insufficient

23. Consider an infinitely long current carrying cylindrical straight wire having radius 'a'. Consider two points A and B, one
inside the wire (point A) and other outside the wire(point B) at a distance of r1 and r2 from the axis. If at both A and B
the magnetic field is one third of that at the surface of the wire, then find the ratio of r1 and r2.
(a)  1/3
(b)  1/6
(c)  1/9
(d)  1/12

24. A solid sphere of radius 𝑅 has uniform charge density 𝜌. If a sphere of radius R/2 concentric with original sphere is
given some extra charge so that total charge density for this spherical region is 2𝜌, then find the electric field at the
point A.

3ρR
(a)   8ε
0
3ρR
(b)   5ε
0
3ρR
(c)   7ε
0
3ρR
(d)   10ε
0

25. In a long cylindrical wire of radius R, magnetic induction varies with distance from the axis as B = C/ rα. Here C and α
are constants and r is the distance from the axis. Then
(a)  Current flowing the cylindrical wire of radius r is proportional to r − α
(b)  Current flowing the cylindrical wire of radius r is proportional to rα − 1
( 1 − α ) Cr ( − α − 1 )
(c)  Current density J as a function of r is 
μ0
( 1 − α ) Cr ( − α − 1 )
(d)  Current density J as a function of r is 
2μ 0

26. In an AC series LCR circuit, the value of resistance is R = 120 Ohms. Resonance frequency is 4000 radians/sec. At
resonance, voltage across the resistor is 60 volts and across the inductor is 40 volts. Now consider the following
statement, and then choose the correct option:
Statement P - Value of inductance is 20mH
Statement Q - Value of capacitance is 3.125µF
Statement R - Value of RMS current is 1 Ampere
Statement S - Value of voltage across capacitor is 40 volts
(a)  Only P, Q and R are correct
(b)  Only P, Q and S are correct
(c)  Only P, R and S are correct
(d)  Only Q, R and S are correct
27. A concave mirror of focal length 40cm and a convex mirror of focal length 5cm are kept coaxially. The object O is kept
at a distance of 60 cm from the concave mirror on its axis as shown in the figure. The image formed after 2 successive
reflections coincides with the object. Then, (Consider paraxial rays only)

(a)  The value of d if the first reflection happens on concave mirror is 25 + 5√43


(b)  The value of d if the first reflection happens on concave mirror is 25 − 5√43
(c)  There is only a single possible value of d
There are two distinct values of d, one for each case whether the first reflection happens on concave mirror or
(d)  
convex mirror

28. In a charging R-C circuit connected to a DC source, let Q1 be the charge that capacitor acquires in time Δt and Q2 be
the charge that the same capacitor acquires in the next equal time interval Δt
For the same capacitor let Δq be the charge that capacitor acquires in time interval t1 and the next equal charge Δq is
acquired by the capacitor in time interval t2. Consider the following statements and then choose the correct option:
Statement P: Q1 < Q2
Statement Q: t2 < t1
Statement R: Q1 > Q2
Statement S:t2 > t1
(a)  Only P and Q are correct statements
(b)  Only R and S are correct statements
(c)  Only P and S are correct statements
(d)  Only R and Q are correct statements

29. Which of the following statements can explain the particle nature of light? Read the statements and then choose the
correct options:
Statement P: A low intensity beam causes photoelectric effect after a substantial time gap in comparison to the high
intensity beam
Statement Q: A high intensity beam or a low intensity beam of same suitable frequency can cause emission of fastest
electrons with almost equal kinetic energy
Statement R: A low intensity beam or a high intensity beam of suitable frequencies causes almost instantaneous
emission of photoelectrons
Statement S: A high intensity beam will always cause photoelectric effect
(a)  Only P and Q are correct
(b)  Only Q and R are correct
(c)  Only Q and S are correct
(d)  Only P, Q and R are correct

30. : Initially the 900μ F capacitor is charged to 100V and the 100μF capacitor is uncharged in the figure shown. Then the
switch S2 is closed for a time t1, after which it is opened and at the same instant switch S1 is closed for a time t2 and
then opened. It is now found that the 100μFcapacitor is charged to 300V. Assume π 2 = 10.

(a)  Minimum possible value of t1 is 3/20 sec


(b)  Minimum possible value of t1 is 3/10 sec
(c)  Minimum possible value of t2 is 1/100 sec
(d)  Minimum possible value of t2 is 1/160 sec
31. Total moles of the gases formed on passing 1 F of charge during electrolysis of aqueous sodium chloride solution is?
(a)  1 mole
(b)  0.5 mole
(c)  2 moles
(d)  0.25 mole

32.
How many statements is/are correct?
(i) Sulphur solution is an example of macro molecular colloid.
(ii) Activated charcoal can be used as an adsorbent in gas masks.
(iii) AgI is used for producing artificial rain by seeding because AgI has crystal structure similar to ice.
(iv) Alums purify muddy water by coagulation of colloidal sized clay & sand.
(v) Physisorption occurs because of van der Waals forces.
(vi) More easily liquefiable gases are adsorbed readily. (For physisorption)
(vii) Under high pressure it results into multimolecular layer on adsorbent surface. (For physisorption)
(viii) Enthalpy of physisorption adsorption (∆ Hadsorption) is low and positive.

(a)  6
(b)  7
(c)  8
(d)  5

33. Among the given elements, which element would form an oxyacid on reaction with nitric acid?
(a)  Pb
(b)  P4
(c)  Zn
(d)  Ag

34.

​Identify the INCORRECTLY matched option.

(a)  Step 1 – Magnetic separation


(b)  Step 2 – carbon reduction
(c)  Step 3 – Liquation
(d)  Step 4 – Liquation
35.
Arrange the following compounds in decreasing order of SN1 reaction,

(a)  I > IV > III > II


(b)  II > III > IV > I
(c)  IV > III > II > I
(d)  III > II > I > IV

36. Which of the following is an example of trans esterification reaction?

(a)  

(b)  

(c)  

(d)  
37. The major product of the acetylation of salicylic acid with Ac 2O / H ⊕ followed by heating with anhydrous AlCl 3 is:

(a)  

(b)  

(c)  

(d)  
38.

Product (Z) is

(a)  

(b)  

(c)  

(d)  

39. Which of the following oxides are neutral, acidic and basic respectively?
(a)  CO2, CO, C3O2
(b)  N2O, N2O3, N2O5
(c)  NO, N2O3, CrO
(d)  SO2, SO3, CO2

40. Which of the above labeled O-atom is able to give major product?

(a)  1
(b)  2
(c)  3
(d)  4
41.

The product will be:

(a)  

(b)  

(c)  

(d)  

42. If 6 ml of 0.1 M AgNO3 is required for complete precipitation of chloride ions present in 30 ml of 0.01 M solution of
CrCl3.5H2O as silver chloride. Find out the formula of the complex
(a)  [Cr(H2O)5Cl]Cl2
(b)  [CrCl2(H2O)4]Cl.H2O
(c)  [CrCl3 (H2O)3]2H2O
(d)  [Cr(H2O)5Cl3]

43. If ∆H of a reaction is positive and k1 and k2 be the rate constants of forward reaction and backward reaction,
respectively, at temperature tºC, k1' and k2' be the respective rate constants at (t + 10)ºC then
k 1′ k1
(a)   =
k 2′ k2
k 1′ k1
(b)   <
k 2′ k2
k 1′ k1
(c)   >
k 2′ k2
(d)  None of these

44. The melting points of Cr, Mn, Fe, Co, Ni in oC is respectively:


(a)  1453, 1244, 1535, 1495, 1903
(b)  1903, 1244, 1535, 1453, 1495
(c)  1903, 1244, 1535, 1495, 1453
(d)  1453, 1244, 1495, 1535, 1453

45. Which of the following will not show yellow ppt with sodium hypoiodite?
(a)  (CH3)2CH–OH

(b)  

(c)  CH3CH2OH

(d)  
46. In a water-ethanol system, the constant boiling mixture has a composition of 95.6% of ethanol by weight and boils at
78.13oC under a pressure of one atm. Which among the following is not true for this water-ethanol constant boiling
mixture?
(a)  This is called azeotropic mixture.
(b)  If composition of mixture is in between pure water and 95.6% ethanol is distilled, pure ethanol will be recovered.
for the composition in between pure water and 95.6% ethanol, when distilled, pure water as a residue will be
(c)  
recovered.
for the composition in between pure ethanol and 95.6% ethanol, when distilled, it is not possible to recover pure
(d)  
water.

47. Which of the following order(s) of properties given is/are correct?


I. Tendency of alkyl halides to undergoes dehydrohalogenation in Williamson, synthesis Follows the order 3o > 2 o > 1
o

II. Propanol < 2 - methyl - 1 - Propanol < 2 - butanol < 2 - Methyl - 2 Propanol
(rate of reaction with HBr)
III.

IV. Toluene < m-dichlorobenzene < o-dichlorobenzene < p-dichlorobenzene – (Order of dipole moment).
Select correct option:
(a)  I, II, IV, are correct
(b)  IV, I are correct
(c)  II, I, IV, are correct
(d)  I, II, and III are correct

48. [Pt(gly)ClBr]+1 has 'n' number of geometrical isomers. Then the spin only magnetic moment and crystal field
stabilisation energy (CFSE) of [Ni(NH3)6]+n respectively: [Ignore pairing energy]
(a)  1.73 BM, –2.0 ∆0
(b)  2.84 BM, –2.0 ∆0
(c)  2.84 BM, –1.2 ∆0
(d)  0 BM, –1.2 ∆0

Δ
49.
PCl5 + P4O10 → oxygen containing compound (X)
How many moles of an oxy acid are formed on hydrolysis of 1 mole (X)
(a)  0
(b)  1
(c)  4
(d)  3
50.

The product 'C' can be:

(a)  

(b)  

(c)  

(d)  

51. Acidic strength order of the following compounds is:


(i) Oxalic acid
(ii) Malonic acid
(iii) Heptanedionic acid
(a)  iii > ii > i
(b)  ii > iii > i
(c)  i > iii > ii
(d)  i > ii > iii

52.

The product B and C are:


(a)  Benzaldehyde and acetaldehyde
(b)  Benzoic acid and acetic acid
(c)  Phenol and propinoaldehyde
(d)  Phenol and acetone

53. When we try to form enol form of following carbohydrates D-glucose, D-galactose, D-mannose and D-fructose. Which
of the following carbohydrates will produce the same enol forms?
(a)  D-glucose & D-fructose
(b)  D-galactose & D-mannose
(c)  D-galactose & D-glucose
(d)  Both (a) and (b)
54.

(a)  

(b)  

(c)  

(d)  

55. Which of the following oxides is not amphoteric in nature?


(a)  Cr2O3
(b)  CrO
(c)  Sb2O3
(d)  ZnO

56. For a multistep reaction, the net reaction is as follows


k
P → Products
where E a = 400 kJ/mol, E a = 180 kJ/mol, E a = 160 kJ/mol are activation energies of k1, k2 and k3 respecively. 

1 2 3

The total rate constant is given by equation k =


( )
k 1k 2 3 / 2
k3

What is the activation energy (in kJ) for the overall reaction?
.

(a)  630 kJ/mole
(b)  500 kJ/mole
(c)  690 kJ/mole
(d)  700 kJ/mole

57. Radius of cation = 120 pm, radius of anion = 140 pm (given = CsCl type structure). Calculate what will be the nearest
distance between cations?
520
(a)   3 pm

670
(b)   3 pm

610
(c)   3 pm

(d)  None
58. The salt which produces brown gas when treated with dilute H2SO4 is:
(a)  NaNO2
(b)  Na2CO3
(c)  NaNO3
(d)  Na3PO4

59. Incorrect property of ozone is


(a)  Powerful oxidising agent
(b)  Bleaching agent
(c)  Non poisonous
(d)  Bent shape

60. The vapour pressure of an aqueous solution is found to be 750 torr at certain temperature 'T’. If 'T' is the
temperature at which pure water boils under atmospheric pressure and same solution show elevation in boiling
point ∆ Tb = 1.04 K, find the atmospheric pressure? (Kb = 0.52 K kg mol–1, Consider solute to be non-electrolytic)
(a)  777
(b)  779
(c)  782
(d)  746

[ ]
61. a b
If A = ; a, b, c, d ∈ {1, 2, 3, 4, 5} and det (A) = 5, then number of possible matrices A is:
c d
(a)  8
(b)  16
(c)  23
(d)  25


62. 1
[ sin x ] + 2

Let [x] denotes the greatest integer ⩽ x, where x ∈ R. If the domain of the real valued function f(x) = is
√16 − x2
(− a, − b] ∪ [c, b], where a, b, c ⩾ 0, then the value of a + b + c is
π
(a)  4 +
2
(b)  2 + π
π
(c)  2 +
2
(d)  4 + π

63.
a = 2 î + ĵ − 2k̂ and ˉ
Let ˉ b = î + ĵ . If ˉ
c is a vector such that ˉ
a. ˉ
c= |ˉc |, |ˉc − ˉa | = 2√2 and the angle between (ˉa × ˉb )
|ˉa ⋅ (ˉb × ˉc ) |2 is
π
and ˉ
c is , then the value of
6
3√3
(a)  
2
9
(b)  
4
27
(c)  
4
3
(d)   2

64. 1 1 2π
The number of real roots of the equation tan − 1x + cos − 1 + cot − 1 = is
x 3
√ 1 + x2

(a)  1
(b)  2
(c)  4
(d)  0
65. a, ˉ
Let ˉ b, ˉ
c be three mutually perpendicular vectors of the same magnitude and ˉ
a is inclined at an angle θ with the
vector 2ˉ ˉ
a+b−ˉ 2
c . Then 36cos 2θ is equal to
(a)  24
(b)  12
(c)  6
(d)  none of these

66.
( )
Let f be a function such that f(x + y) = f(x) + f(y) for all x & y,  and f(x) = 2x 2 + 3x g(x) for all x,  where g(x) is a continuous
function and g(0) = 9 .

Then f ′(x) is equal to


(a)  9
(b)  3
(c)  27
(d)  6

()
67.
an
(
If a n and b n are positive integers and a n + √2b n = 2 + √2 ) n, then nlim
→∞
bn
=

(a)  √2
(b)  2
(c)  e √2
(d)  e 2

π
68.
2
If [x] denotes the greatest integer less than or equal to x, then the value of the integral ∫ [[x] − cosx]dx is equal to:
π

2
π
(a)  −
2
π
(b)  
2

(c)  0


(d)  −
2

[ () ( ) ( )]
69.
1 5 10 10 ( n − 1 )
If f : R → R is given by f(x) = x 2 + 1, then the value of lim n f(0) + f n + f n + . . . . + f n
, is:

n→∞
206
(a)  
3
(b)  102
(c)  202
7
(d)  
2

70. From bag A having 2 Red, 1 Black balls and from bag B having 3 Red, 2 Black balls, 1 ball each is transferred to bag C
which was initially empty. Now from Bag C, 1 ball is drawn, then probability of this ball to be Red is
2
(a)  
3
19
(b)  
30
17
(c)  
30
(d)  None of these
71. If for some k ∈ R, the following system of linear equations

3x − y + 4z = 3,

x + 2y − 3z = − 2,

6x + 5y + kz = − 3,

has infinitely many solutions, then for that k, value of 7x + 7y − 8z =


(a)  3
(b)  − 5
(c)  5
(d)  − 3

72. If f : {1, 2, 3, 4, 5, 6} → {1, 2, 3, 4} is a function with range 


{1, 2, 3, 4}, then number of such functions f such that f(1) < f(2)
is 22 × 3 ! × a, where a =
(a)  4
(b)  5
(c)  6
(d)  none of these

73. Inside a frustum of right circular cone of radii 10 cm and 15 cm and height 12 cm, a right circular cylinder whose axis

is same as that of the frustum is inscribed. Then the maximum possible volume of cylinder in cm 3 is ( )
(a)  1200 π
(b)  1500 π
(c)  1800 π
(d)  2100 π

74. Four dice are thrown simultaneously and the numbers shown on these dice are recorded in a 2 × 2 matrix. The
probability that such formed matrix has exactly 2 different entries and is non-singular is
35
(a)  
216
25
(b)  
216
5
(c)  
72
5
(d)  
36

75. a, ˉ
Let ˉ b and ˉ a + 7ˉ
c be three vectors of equal magnitude such that 4ˉ c=ˉ
b + 9ˉ 0. Then if α, β, γ are angles between ˉ
b&
ˉ
c, ˉ
c&ˉ a and ˉ ˉ then
a &b
(a)  α, β and γ are acute
(b)  γ is acute and α, β are obtuse
(c)  α is obtuse and β, γ are acute
(d)  α, β and γ are obtuse.

76. Consider the following three equations:


(2k − 9)x 2016 + 3y2016 + (3k − 15)z 2016 = 0

− 5x 2016 + ky2016 − 8z 2016 = 0


(− k + 2)x 2016 − y2016 + (− k + 3)z 2016 = 0

Then number of k for which the above system of equation possesses non-zero real solution is
(a)  0
(b)  1
(c)  2
(d)  3
77. √
sin − 1 x 2 − x + 1
If the domain of the function f(x) = is the interval [α, β), then α + β is equal to :

√ cos − 1 ( )
2x − 1
2
−2
π

3
(a)  
2
(b)  2
1
(c)  
2
(d)  1

[ ]
78.
min ( sin x , { x } )
If α is a root of the equation sinx + 1 = x then lim

x−1
x→α
( where [ . ] denotes greatest integer function, {x} is fractional part of x )
(a)  is equal to −1
(b)  is equal to 0
(c)  is equal to 1
(d)  does not exist

79. →
( ) →
( )
Let L be the line of intersection of planes r. î − ĵ + 2k̂ = 2 and r. 2 î + ĵ − k̂ = 2. If P(α, β, γ) is the foot of
perpendicular on L from the point (0, 0, 0), then the value of 35(α + β + γ) is equal to :
(a)  42
(b)  45
(c)  56
(d)  65

| ()|
80.
y π
Area of the region in which point p(x, y), {x > 0} lies; such that y ⩽ √16 − x 2 and tan − 1
x

3
is

(a)  
( ) 16
3
π

(b)  
( √)8π
3
+8 3

(
(c)   4 3 − π
√ )
(d)   (√3 − π )

()
81.
sec 2x − 2010 P(x) π
If ∫ dx = + C, then value of P is

sin 2010x sin 2010x 3

(where C is a constant of integration)


(a)  0
1
(b)   3

(c)  √3
(d)  None of these
( ) ( )
82.
x x
The solution of the differential equation xdy y2e xy + e y = ydx e y − y2e xy , is

(where λ is a constant of integration)

(a)  xy = ln e x + λ ( )
= ln (e + λ )
2
x xy
(b)  
y

(c)  xy = ln e y + λ
( ) x

(d)  xy2 = ln e xy + λ( )
83. Fifteen coupons are numbered 1, 2, 3, . . . .15. Seven coupons are selected at random one at a time with replacement.
The probability that the largest number appearing in the process is 9, is

(a)  
() 9
16
6

(b)  
() 8
15
7

(c)  
()
3 7
5

(d)  none of these

84. Through a point P(h, k, l) a plane is drawn at right angles to OP to meet the co-ordinate axes at A, B and C. If OP = p,
where O is origin, then the area of ΔABC is
p 2hk
(a)  
l2
p 3l
(b)  
3hk
p 2l 2
(c)  
2hk
p5
(d)  
2hkl

85. If a real valued function f satisfies f(10 + x) = f(10 − x) and f(20 − x) = − f(20 + x), for all x ∈ R , then which of the
following statements is true?
(a)  f is an even function
(b)  f is an odd function
(c)  f is a constant function
(d)  f is a non-periodic function

( ) ( )
86.
x d 2y dy m
If y = xlog , then x n = x −y , where:
a + bx dx 2 dx

(a)  n = 3 , m = 2
(b)  n = 2 , m = 3
(c)  n = m = 2
(d)  n = m = 3
{[
87.
sin [ x ]
α+ ; x>0
x
2 ; x=0
f(x) = , where [.] is G.I.F.

β+
sin x − x
x3 ] ; x<0

If f(x) is continuous at x = 0 , then β − α is equal to


(a)  1
(b)  −1
(c)  2
(d)  −2

88. π/4 π/4 π/4


Let I1 = ∫ x 2008(tanx) 2008dx, I2 = ∫ x 2009(tanx) 2009dx and I3 = ∫ x 2010(tanx) 2010dx. Then the correct order sequence,
0 0 0
is
(a)  I2 < I3 < I1
(b)  I1 < I2 < I3
(c)  I3 < I1 < I2
(d)  I3 < I2 < I1

89. If the tangent to the curve x = 1 − 3t 2, y = t − 3t 3 ( t is parameter) at the point P(−2, 2) meets the curve again at Q, then
the angle between the tangents at P and Q is
π
(a)  
6
π
(b)  
4
π
(c)  
3
π
(d)  
2

90. A lamp of negligible height is placed on the ground l 1 metre away from a wall. A man l 2 metre tall is walking at a speed
l1
of 10 m/sec from the lamp to the nearest point on the wall. When he is mid-way between the lamp and the wall, the
rate at which the length of his shadow on the wall is changing is
− 5l2
(a)   m/sec
2
− 2l2
(b)   m/sec
5
− l2
(c)   m/sec
2
− l2
(d)   m/sec
5
Answer Key

1. A 2. A 3. A 4. C 5. B 6. C


7. C 8. B 9. A 10. A 11. C 12. D
13. C 14. B 15. D 16. B 17. A 18. D
19. A 20. B 21. A 22. C 23. C 24. A
25. C 26. B 27. C 28. B 29. B 30. A
31. A 32. A 33. B 34. D 35. B 36. C
37. D 38. C 39. C 40. A 41. C 42. A
43. C 44. C 45. D 46. B 47. D 48. C
49. B 50. D 51. D 52. D 53. A 54. D
55. B 56. A 57. A 58. A 59. C 60. A
61. C 62. D 63. C 64. B 65. D 66. C
67. A 68. D 69. A 70. B 71. B 72. B
73. A 74. B 75. B 76. A 77. C 78. D
79. C 80. B 81. C 82. C 83. D 84. D
85. B 86. A 87. A 88. D 89. D 90. B
Solutions

1.  (A)

Kα photon is less energetic than Kβ photon as k α corresponds to 2 → 1 transition while k β corresponds to 3 → 1 transition

hC
λ cutoff
= eV 0  (hence λ cutoff has maximum energy)

2.  (A)

B = Magnetic field between tapes

μ0 ( I / b )
= 2
×2

μ 0I
=
b

Consider a region like ABCD

ϕ ABCD = B × h × 1

= LI
μ 0I
Bh b
×h
⇒ L= I = I

μ 0h
⇒ L= b

(Option A is correct)

Alternatively, 

1
after writing B, we can calculate energy between tapes in unit length & equals to  2 LI2

B2 1
⇒ 2μ hb = 2 LI2
0

( μ 0I / b ) 2 1
⇒ 2μ 0
hb = 2 LI2

μ 0h
⇒ L=
b
3.  (A)
Charges will be induced as shown
q
[
V 0 = 4πε r + 2R − R
0
1 1 1
] q
= 4πε r − 2R
0 [ 1 1
]

4.  (C)
2π 2π
ΔP = Δx = [(μ − 1)t]
λ λ

∴ Inet = 4I0cos 2
ΔP
2 ( )
π
= 4I0cos 2 (μ − 1)t
λ [ ]
5.  (B)
sin 60 ∘ sin 60 ∘
sin r
= μ ⇒ sin r = μ

1
= 2 ⇒ r = 30 ∘

AI = AN + NI = (R – R cos 60°) + h cot 30° = 2R

6.  (C)
7.  (C)

2Tsin 2 ( )

= BiRdθ

For small angle

sinθ ≈ θ

Tdθ = BiRdθ

T = BiR

2πR = L

L
R= T

BiL
= 2π

8.  (B)
According to de Broglie
2πr = nλ
⇒ 2πr3 = 3x

⇒ 2πr5 = 5λ

r3 3x
∴ r = 5λ

3 9r
Now, since r ∝ n 2, therefore r = 25
5

3 x
⇒ =

5 λ

5x
λ=

3
9.  (A)

F AB = 0
F CD = 0
F AD ≠ 0 & F BC ≠ 0
F net ≠ 0

View from z-axis

→ →
The resultant of F DA \&  F BC will be along x-axis

τ = μ×B
μ0I 4μ0I2a 2
= (2I)(2a) 2 × =
2πb πb
Since net flux passing loop is zero, therefore M is zero
10.  (A)

By theory
11.  (C)

R = resistance
P = Power
V = Voltage
η= percentage efficiency
I = P/V
P′ = Power dissipated
P2
= P ′ = I2R = R

V2
P P
η= × 100 = × 100

( )

P+P
P 2R
P+
V2


1 RPη
= × 100 ⇒ V =

( )
(100 − η)
RP
1+
V2

12.  (D)

Let ρ M= Mass density

ρ e = charge density

V = Volume of sphere
dm = mass of element
M
= ρ Mdv = V dV

And
Q
dq = charge of element = ρ edV = dV

V
Electric force between sphere and element

=
K(Q − dq)dq
=
( Q−Q V )
dV QdV
V

R2 R2

=
KQ 2 1 − V( ) dV dV
V

R2
Similarly, gravitation force between element and sphere is

=
G(M − dm)dm
=
(
G M − V dV
M
) MdV
V

R2 R2

=
GM2 1 − V
( )( ) dV dV
V

R2
Given that KQ2 = GM2
⇒ Electric force = Gravitational force

⇒ Net force of interaction is zero


13.  (C)

( )
Imax R 1 + G = 1V. . . . . (1)

( R 1R 2

)
Imax G + R + R = 0.2 V. . . . (2)

1 2

On solving (1) and (2)


We obtain
[
R1 R1 − 4G ]
R2 =

4 [ R1 + G ]
Since resistance cannot be –ve,
∴ R 1 > 4G

14.  (B)

Since magnetic field is along y axis and velocity of electron in also along y axis, therefore no magnetic force will act on
electron.
Since EM wave propagates along x axis and magnetic field is along y axis, therefore the electric field will be along z – axis .
Hence electric force will be along z – axis.
E = CB = 3 × 10 8 × 10 − 7 = 30 V/m

Felectric max = q Emax = 1.6 × 10–19 × 30 = 4.8 × 10–18 N

15.  (D)

e= | |l di
dt

| 10 × 10 − 3 ×
0.5 × 10 − 3
0.02 |
5 × 10 − 6
= = 2.5 × 10 − 4 = 0.25 mV
0.02

16.  (B)
Magnetic field cannot do work so statement 2 is wrong.
→ →
∵ the direction of V × B at P is along positive y, but particle deflects towards negative y, so we can say that q must be
negative.
∵ Kinetic energy has increased.
Hence we could say that electric field has done positive work.
17.  (A)
For first wave

B 1 = B 0cos(ky − ωt) î

Wave moves along y axis and magnetic field is along x axis


→ → →
Hence E must be along z direction as E × B gives direction of wave and E0 = CB0


So E 1 = cB 0(ky − ωt) k̂


Now Similarly for B 2 = B 0cos(kx − ωt) k̂


We have E 2 = CB 0cos(kx − ωt) ĵ

( )
→ → → → →
F = q E 1 + E 2 + v→ × B 1 + v→ × B 2
Hence
[ (
= q CB 0cos(ky − ωt) k̂ + CB 0cos(kx − ωt) ĵ + 0.5C ĵ × B 0cos(ky − ωt) î + B 0cos(kx − ωt) k̂ )]
Putting t = 0 and x,y,z = 0

[ (
F = q CB 0k̂ + CB 0 ĵ + 0.5C ĵ × B 0 î + B 0k̂ )]
[
= qCB 0 k̂ + ĵ +
1
2
1
( )
− k̂ + î
2 ]
= qCB 0

2 [
+ ĵ +

2 ]
18.  (D)

By applying Nodal Analysis


x − 10 x − 0 x − 10 x − 0
+ + + =0
5 3 3 5
2x − 10 10 − 2x
⇒ =
5 3
⇒ 6x − 30 = 50 − 10x
⇒ 16x = 80 ⇒ x = 5
So
10 − x
i1 = = 1 Ampere
5
x−0 5
i2 = =  Ampere
3 3
So current in wire BC is 2/3 Ampere from C to B.
19.  (A)

2h 4h
= +
3μ1 3μ2
2h 4h 5h 10h
= + = =
3×3 3 × 1.5 4.5 9

20.  (B)
h h
λ= =
P
√2mK
h2
⇒ Ki =
2λ 21m
h2
⇒ Kf =
2(.4λi) 2m
2
Kf h2 2 λi m
1 25
⇒ = = =
( )
Ki m h2 .16 4
2 0.4λ i 2

21.  (A)
mv
r=
qB
√2mK
⇒r=
qB

⇒K=
r2q 2B 2
=
(
(0.02) 2 × 1.6 × 10 − 19 ) (2 × 10 ) Joule
2 −4 2

2m 2 × 10 − 30
0.00128 × 10 − 27 1.28 × 10 − 30
⇒ K in (eV) = = = 1.28eV
10 − 30 10 − 30
hc
Now, λ = ϕ + K max = 2 + 1.28 = 3.28 eV

12400 ∘
So λ = 3.28 = 3780A

22.  (C)
→ →
∵ p. r = 0

So this point must be at equatorial position of the dipole.


Hence electric field at this point must be anti-parallel.
23.  (C)

Let current i is passing through wire.


μ 0i
Now magnetic field at surface B S = 2πa

So Magnetic field at B
μ0i μ0i
BB = =
6πa 2πr2
⇒ r2 = 3a
Magnetic field at A can be found using Ampere’s law :
i
B A2πr1 = μ0 2 πr21

πa
μ 0ir 1 μ 0i
BA = = 6πa

2πa 2
a
r1 =

3
a
r1 3 1
So, r = 3a = 9
2

24.  (A)
We can assume two spheres, one of radius R and other of R/2 kept at C as centre.

ρR
Electric field due to outer sphere at A = 3ε

Electric field due to smaller sphere at A =


k3π
4
()
R
2
3
ρ
=
ρR

R2 24ε 0
ρR ρR
Net Electric field 3ε + 24ε

0 0
9ρR 3ρR
= 24ε ⇒ 8ε
0 0
25.  (C)
∫ B ⋅ dl = μ0I
⇒ Cr − α(2πr) = μ0I
2πC r ( − α + 1 )
or I =

μ0
2πC r ( −α+1)

∫ J(2πrdr) =

μ0
upon differentiating,
( − α + 1 ) Cr ( − α − 1 )
or J = μ0

26.  (B)
i rms = 60 / 120 = 0.5
X C = X L = 40 / i rms = 80
Now we can find both L and C

27.  (C)
It can be seen from the ray diagram that if the image coincides with the object after two reflections, then the order of
mirrors does not matter.

For concave mirror


1 1 1 1 1 1
+ = ⇒ = − ⇒ v = − 120

v − 60 − 40 v 60 40
For convex mirror
1 1 1
−d
+ 120 − ( 60 + d ) = 5

Solving we get d = 25 + 5√37

28.  (B)
As the capacitor charges, the rate of charging i.e. the current decreases

29.  (B)
A is an incorrect statement, if it was true then it will support wave theory.
B and C are standard facts which support particle theory.
In option D, a high intensity beam may not cause a photoelectric if its frequency is less then threshold frequency.

30.  (A)
It can be seen that all the energy present in 900 microfarad capacitors has reached the 100 microfarad capacitor. Hence, t1
will have a minimum value of T1/4 since in this much time, all the energy from the capacitor will reach the inductor.
Similarly, t2 will have a minimum value of T2/4.

31.  (A)
H 2O
NaCl(aq) → Na + (aq) + Cl − (aq)

1
Cathode: H 2O(l) + e − → 2 H 2(g) + OH − (aq)

1
Anode: Cl − (aq) → 2 Cl 2(g) + e −

1 1
Net reaction: NaCl(aq) + H 2O(l) → Na + (aq) + OH − (aq) + 2 H 2(g) + 2 Cl 2(g)

1 F charge will produce half mole of hydrogen gas at cathode as well as half mole of chlorine gas at anode. Thus, total
moles of gases formed is 1 mole.
32.  (A)
(i) Sulphur solution is an example of multimolecular colloid.
(viii) Enthalpy of adsorption in physisorption is negative ∆ G = ∆ H – T∆ S
As the entropy decreases (∆ S = –ve) the ∆ H must be negative having a high magnitude. Therefore, the spontaneous
adsorption will have negative enthalpy change.

33.  (B)
1
P
2 4
+ 10HNO 3 → 2H 3PO 4 + 10NO 2 + 2H 2O

34.  (D)

Step 4 should be poling.

35.  (B)
More the stable carbocation more will be the rate of reaction.
Stability of carbocation is govern by hyperconjugation effect, inductive effect and resonance effect.
In second II three alpha hydrogens are present hence its shows highest hyperconjugation effect.
In compound IV inductive effect is applicable only.
Hence correct order is II > III > IV > I

36.  (C)
Trans esterification reaction is the conversion of one ester in to another ester. Only reaction in option (c) is trans
esterification reaction.

37.  (D)
38.  (C)

39.  (C)
Acidic oxides: Oxides that react with water to form an acids
Basic oxides: Oxides that react with water to form bases
Neutral oxides: Oxides which show neither basic nor acidic behavior on reacting with water
In option (c)
N2O → Neutral oxide
N2O3 → Acidic oxide (N2O3 + H2O → H2N2O4 ≡ 2(HNO2) ⇒ It is an acid)
CrO → Basic oxide (CrO + H2O → Cr(OH)2 ⇒ It is a base)
Hence answer is (c)

40.  (A)
In case of 1st number oxygen atom after removal of H2O back bonding is present.

41.  (C)

42.  (A)
mole of CrCl3.5H2O = M × V = 0.3 m mol.
Moles of AgNO3 = M × V = 0.6 m mol.
i.e., we have 0.6 m mol. of Ag+ which require 0.6 m mol. of Cl– for complete precipitation since, we have 0.3 m mol. of
CrCl3.5H2O. Therefore, each such molecule will have to furnish two Cl– ions.
Hence, the formula of complex is [Cr(H2O)5Cl]Cl2
Answer is (a)
43.  (C)

k1′
k1
= e  - 
( Ea ) f
R ( 1
T + 10
1
−T )
k2′
k2
= e  - 
( Ea ) b
R ( 1
T + 10
1
−T )

()()
k1′
k2′
×
k2
k1
= e R ( T ) ( T + 10 ) ( ( Ea ) f − ( Ea ) b )
1

k1′ k1 10ΔH
= × e R ( T ) ( T + 10 )
k2′ k2
k 1′ k1
Then, > k

k 2′ 2

Hence answer is (c)

44.  (C)
More number of unpaired electrons more will be melting point
Mn have half field electronic configuration (stable)
So, it will show weak metallic bonding so shows less melting point
Cr → 3d5 4s1 ⇒ 6e–
Mn → 3d5 4s2 ⇒ 5e– (stable configuration)
Fe → 3d6 4s2 ⇒ 4e–
Co → 3d7 4s2 ⇒ 3e–
Ni → 3d8 4s2 ⇒ 2e–
Therefore, answer is (c)

45.  (D)
Ethanol, secondary al-2-ol which on oxidation gives acetyl ketone can show haloform

46.  (B)
When between azeotrope and water, water if higher boiling so left behind and vapor will be azeotrope composition

47.  (D)
(i) bulky substrate with base gives alkene in williamson synthesis
(ii)

Greater the stability of R⊕ (Intermediate) Greater will be reactivity of alcohol.


(iii) Basic nature ∝ +I +M group
I

− I . − M group
(iv) Dipole moment of p-dichlorobenzene is zero.
48.  (C)
[Pt(gly)ClBr]+1 has 2 Geometrical isomer
Ni+2 → 3d8

μ = √2(2 + 2) = √8 = 2.83 B.M.

d8 configuration of Ni+2 hence CFSE will be


2 3 6
6 × − Δ 0 + 2 × Δ 0 = − Δ 0 = − 1.2Δ 0
5 5 5

49.  (B)
Δ
6PCl 5 + P 4O10 → 10POCl 3
(X)
POCl 3 + 3H 2O → 3HCl + H 3PO4
(X)

50.  (D)

51.  (D)
Due to the presence of more +I Groups the malonic acid and heptanedionic acid are less acidic than oxalic acid.

52.  (D)
H 3PO 4 ⊕
CH 3 − CH = CH 3 → CH 3 − CH − CH 3

53.  (A)
All 3 D-glucose, D-fructose, and D-mannose will produce the same enol form.
54.  (D)

55.  (B)
CrO + H2O → Cr(OH)2
So, CrO is a basic oxide

Hence, CrO is not an amphoteric oxide

56.  (A)

Ae  - E a / RT =
[ Ae  - E a3 / RT ]
Ae  - E a1 / RT × Ae  - E a2 / RT 3 / 2

[ ( ) ]
= Ae  - E a 1 - E a 2 + E a 3 / RT
3/ 2

3
E a = 2 [400 + 180 − 160]

E a = 630 kJ/ mole

57.  (A)
a√ 3 a√ 3
rc + ra = ⇒ 260 pm =
2 2
520
a= pm
√3
58.  (A)

2NaNO2 + H 2SO4 → Na 2SO4 + NO( ↑ ) + NO2( ↑ ) + H 2O


( dil . ) ( Brown gas )

59.  (C)
Ozone is poisonous in nature.

60.  (A)
1.04 = 0.52 × m ⇒ m = 2
P 0 − 750 1000
2= × 18

750
0
⇒ P = 777 = Patm
P0 = Patm (because at T water boils)
So, at T : P0 = Patm
because when water boils V.P. become equals to atmospheric pressure
61.  (C)
det (A) = 5

ad − bc = 5

ad = 5 + bc

Exactly one out of ad and bc must be odd.


Possible odd products: 1, 3, 5, 9, 15, 25

Following cases are possible:


bc = 1 = 1 × 1, ad = 6 = 2 × 3 → 2 ways

bc = 3 = 1 × 3, ad = 8 = 2 × 4 → 4 ways

bc = 5 = 1 × 5, ad = 10 = 2 × 5 → 4 ways

ad = 9 = 3 × 3, bc = 4 = 1 × 4 = 2 × 2 → 3 ways

bc = 15 = 3 × 5, ad = 20 = 4 × 5 → 4 ways

ad = 15 = 3 × 5, bc = 10 = 2 × 5 → 4 ways

ad = 25 = 5 × 5, bc = 20 = 4 × 5 → 2 ways

So, total 23 ways.


So, number of possible matrices A is 23.

62.  (D)
16 − x 2 > 0

(x + 4)(x − 4) < 0

x ∈ (− 4, 4)
and [sinx] = 0 or 1

So, sinx ⩾ 0 & x ∈ (− 4, 4)

So, Domain = (− 4, − π] ∪ [0, π]

So, a + b + c = 4 + π

63.  (C)
|ˉc − ˉa |2 = 8

⇒ c 2 + a 2 − 2ˉ
a⋅ˉ
c = 8

⇒ 2
c − 2c + 1 = 0

⇒ |ˉc | = 1

|ˉa × ˉb |2 |ˉc | 2 ⋅ cos 2 6

π
|[ a b c ] | 2 =

=
| |
27
9 3
3 2
⋅ 4

= 4

64.  (B)
Case 1: x > 0

1 1 2π
tan − 1x + cos − 1 + cot − 1 x = 3

√ 1 + x2

⇒ 3tan − 1x = 3


⇒ x = tan 9

Case 2: x < 0

1 1 2π
tan − 1x + cos − 1 + cot − 1 x = 3

√ 1 + x2
1 2π
⇒ cot − 1 =

x 3
⇒ x = − √3

65.  (D)
2 |a
|2 ˉ
2
cosθ = =

| a | ⋅ | a | ⋅ √6 √6
ˉ ˉ

1
cos2θ = 2cos 2θ − 1 = 3

36cos 22θ = 4

66.  (C)

f( x+h ) −f( x)
f ′(x) = lim h

h→0
f( x) +f( h ) −f( x)
= lim h

h→0

( 2h 2 + 3h
)g(h)
= lim

h
h→0
= lim (2h + 3)g(h)

h→0
= 3g(0) = 3(9) = 27

67.  (A)

We have a n + √2b n = 2 + √2 ( ) n

⇒ a n − √2b n = 2 − √2 ( ) n

Therefore a n =
1
2 [( 2 + √2 )n + (2 − √2 )n ]

[( 2 + √2 ) n − ( 2 − √2 ) n ]
and b n =

2√2

an [( 2 + √2 ) n + ( 2 − √2 ) n ]
Therefore = √2

[( ]
bn
2 + √2 ) (
n
− 2 − √2 ) n

= √2
[ ( )]
1+
2 − √2

2 + √2
n

[ ( )]
1−
2 − √2

2 + √2
n

Hence lim
n→∞ () an
bn
= √2 ( ) 1+0
1−0
= √2 ( ∵0<
2 − √2

2 + √2 )
<1

68.  (D)
π
2
∫ [[x] − cosx]dx

π

2
π
2
= ∫ ([x] + [− cosx])dx
π

2
π π
2 2
= ∫ (− [x] − 1)dx + ∫ (− 1) dx

π π
− −
2 2
π π

12 2 3π
= 2 ∫ (− 1)dx + ∫ (− 1) dx = − 2
π π
− −
2 2

69.  (A)
\mathop {\lim }\limits_{n \to \infty } \,\frac{1}{n}\left[ {f\left( 0 \right) + f\left( {\frac{5}{n}} \right) + f\left( {\frac{{10}}{n}}
\right) + .... + f\left( {\frac{{10\left( {n - 1} \right)}}{n}} \right)} \right]
= \mathop {\lim }\limits_{n \to \infty } \,\frac{1}{n}\sum\limits_{r = 0}^{2\left( {n - 1} \right)} {f\left( {\frac{{5r}}{n}} \right)}
= \int\limits_0^2 {f\left( {5x} \right)dx}
= \int\limits_0^2 {\left( {25{x^2} + 1} \right)\,dx} = 25 \times \frac{8}{3} + 2
= \frac{{206}}{3}
70.  (B)
P = \frac{1}{2} \times \frac{2}{3} + \frac{1}{2} \times \frac{3}{5} = \frac{{10 + 9}}{{30}} = \frac{{19}}{{30}}

71.  (B)
3x - y + 4z = 3\,\,\,\,\,\,\,\,\,\,\,\,....\left( 1 \right)
x + 2y - 3z = - 2\,\,\,\,\,\,\,\,\,\,\,....\left( 2 \right)
6x + 5y + kz = - 3\,\,\,\,\,\,\,\,\,\,\,....\left( 3 \right)
\left| A \right| = \left| {\begin{array}{*{20}{c}}   3&{ - 1}&4 \\   1&2&{ - 3} \\   6&5&k \end{array}} \right| = 0
\Rightarrow 6\left( { - 5} \right) - 5\left( { - 13} \right) + k\left( 7 \right) = 0
\Rightarrow k = - 5
Adding (2) & (3)
7x + 7y - 8z = - 5

72.  (B)
To find total number of functions having range \left\{ {1,2,3,4} \right\}, we need to divide 1,2,3,4,5,6 into 4 groups of 1,1,1,3
or 1,1,2,2.
So total number of such functions
= \frac{{6!}}{{\left( {3!} \right)\left( {3!} \right)}} \times 4!\, + \frac{{6!}}{{{{\left( {2!} \right)}^2}\left( {2!} \right)\left( {2!} \right)}}
\times 4!
= 480 + 1080
= 1560
Number of such functions, when f\left( 1 \right) = f\left( 2 \right), is
= \frac{{5!}}{{\left( {2!} \right)\left( {3!} \right)}} \times 4! = 240
Required number of functions
= \frac{{1560 - 240}}{2} = \frac{{1320}}{2} = 660
\Rightarrow 22 \times 3!\, \times a = 660
\Rightarrow a = 5

73.  (A)

\frac{{15}}{H} = \frac{{10}}{{H - 12}} = \frac{r}{{H - h}}


\Rightarrow H = 36
\Rightarrow \frac{r}{{36 - h}} = \frac{{15}}{{36}}
\Rightarrow 12r = 180 - 5h
\Rightarrow h = \frac{{180 - 12r}}{5}
Volume of cylinder, V = \pi {r^2}h
= \pi {r^2}\left( {\frac{{180 - 12r}}{5}} \right)
= \frac{{ - 12\pi }}{5}{r^2}\left( {r - 15} \right)
= \frac{{ - 12\pi }}{5}\left( {{r^3} - 15{r^2}} \right)
\frac{{dV}}{{dr}} = \frac{{ - 12\pi }}{5}\left( {3{r^2} - 30r} \right) = \frac{{ - 36\pi }}{5}r\left( {r - 10} \right)
\frac{{dV}}{{dr}} = 0 \Rightarrow r = 10\,\,\,\,\,\,\left( {\because \,\,r \ne 0} \right)
{\left. {\frac{{{d^2}V}}{{d{r^2}}}} \right|_{r = 10}} = \frac{{ - 12\pi }}{5}\,\left( {60 - 30} \right) = - {\text{ve}}
So, V is maximum for r = 10.
{V_{\max }} = \pi \times 100 \times 12 = 1200\pi
74.  (B)
Let exactly 2 different entries are a and b. Then
(1) 3a,\,\,1b are non-singular
(2) 2a,\,\,2b are non-singular if they are \left[ {\begin{array}{*{20}{c}}   a&b \\   b&a \end{array}} \right] or \left[
{\begin{array}{*{20}{c}}   b&a \\   a&b \end{array}} \right]
P = \frac{{^6{C_2} \times \left[ {2 \times 4 + 2} \right]}}{{6 \times 6 \times 6 \times 6}}
= \frac{{150}}{{{6^4}}} = \frac{{25}}{{216}}

75.  (B)
Let \left| {\bar a} \right| = \left| {\bar b} \right| = \left| {\bar c} \right| = k and 4\bar a + 7\bar b + 9\bar c = 0
4\bar a + 7\bar b = - 9\bar c \Rightarrow \bar a \cdot \bar b = \frac{{2{k^2}}}{7}
\cos \gamma = \frac{{\bar a \cdot \bar b}}{{\left| {\bar a} \right|\left| {\bar b} \right|}} = \frac{2}{7}
\Rightarrow \gamma = {\cos ^{ - 1}}\left( {\frac{2}{7}} \right) = {\text{acute}}
4\bar a + 9\bar c = -7\bar b \Rightarrow \bar a \cdot \bar c = - \frac{2}{3}{k^2}
\cos \beta = \frac{{\bar a \cdot \bar c}}{{\left| {\bar a} \right|\left| {\bar c} \right|}} = \frac{{\bar a \cdot \bar c}}{{{k^2}}} =
\frac{{ - 2}}{3}
\Rightarrow \beta = {\cos ^{ - 1}}\left( {\frac{{ - 2}}{3}} \right) = {\text{obtuse}}
7\bar b + 9\bar c = - 4\bar a \Rightarrow \bar b \cdot \bar c = \frac{{ - 19{k^2}}}{{21}}
\cos \alpha = \frac{{\bar b \cdot \bar c}}{{\left| {\bar b} \right|\left| {\bar c} \right|}} = \frac{{\bar b \cdot \bar c}}{{{k^2}}} =
\frac{{ - 19}}{{21}}
\Rightarrow \alpha = {\cos ^{ - 1}}\left( {\frac{{ - 19}}{{21}}} \right) = {\text{obtuse}}

76.  (A)

For non-zero real solutions,


\left| {\begin{array}{*{20}{c}}   {2k - 9}&3&{3k - 15} \\   { - 5}&k&{ - 8} \\   { - k + 2}&{ - 1}&{ - k + 3} \end{array}} \right| = 0
\Rightarrow \left( {2k - 9} \right)\left[ { - {k^2} + 3k - 8} \right] - 3\left[ {5k - 15 - 8k + 16} \right] + \left( {3k - 15} \right)\left[ {5 +
{k^2} - 2k} \right] = 0
\Rightarrow {k^3} - 6{k^2} + 11k - 6 = 0
\Rightarrow \left( {k - 1} \right)\left( {{k^2} - 5k + 6} \right) = 0
\Rightarrow \left( {k - 1} \right)\left( {k - 2} \right)\left( {k - 3} \right) = 0
\Rightarrow k = 1,2,3

For all the values of k=1, 2 or 3

x^{2016} & z^{2016} have opposite sign.

Hence no real solutions possible.

77.  (C)
(1) {x^2} - x + 1 \leqslant 1
x \in \left[ {0,1} \right]
(2) \frac{{2x - 1}}{2} \in \left[ { - 1,\,\,0} \right)
x \in \left[ { - \frac{1}{2},\frac{1}{2}} \right)
So, domain is \left[ {0,\frac{1}{2}} \right)
78.  (D)

Given equation is
\sin x + 1 = x
\Rightarrow \sin x = x - 1
Its root \alpha can be observed in the following graph :

Clearly, 1 < \alpha < 2


Hence for x \to \alpha \,,\,\,\left\{ x \right\} = x - \left[ x \right] = x - 1
\Rightarrow \mathop {\lim }\limits_{x \to \alpha } \left[ {\frac{{\min \left( {\sin x,\,\,\{ x\} } \right)}}{{x - 1}}} \right] = \mathop
{\lim }\limits_{x \to \alpha } \left[ {\frac{{\min \left( {\sin x,\,\,x - 1} \right)}}{{x - 1}}} \right]
LHL = \mathop {\lim }\limits_{x \to {\alpha ^ - }} \left[ {\frac{{\min \left( {\sin x,\,\,x - 1} \right)}}{{x - 1}}} \right]
When x < \alpha
x - 1 < \sin x
\Rightarrow \min \left( {\sin x,\,\,x - 1} \right) = x - 1
So, LHL = \mathop {\lim }\limits_{x \to {\alpha ^ - }} \left[ {\frac{{x - 1}}{{x - 1}}} \right] = 1
Now, RHL = \mathop {\lim }\limits_{x \to {\alpha ^ + }} \left[ {\frac{{\min \left( {\sin x,\,\,x - 1} \right)}}{{x - 1}}} \right]
When x > \alpha
\sin x < x - 1
\Rightarrow \min \left( {\sin x,\,\,x - 1} \right) = \sin x
So, RHL = \mathop {\lim }\limits_{x \to {\alpha ^ + }} \left[ {\frac{{\sin x}}{{x - 1}}} \right] = 0
( \because \,\,{\text{for}}\,\,x \to {\alpha ^ + },\,\,\frac{{\sin x}}{{x - 1}} is just less than 1 )
Hence,{\text{LHL}} \ne {\text{RHL}}
So, Limit does not exist.

79.  (C)
\alpha - \beta + 2\gamma = 2
\underline {2\alpha + \beta - \gamma = 2}
\alpha = \frac{1}{3}\left( {4 - \gamma } \right),
 \beta = \frac{1}{3}\left( {5\gamma - 2} \right)
and \left| {\begin{array}{*{20}{c}}   \alpha &\beta &\gamma \\   1&{ - 1}&2 \\   2&1&{ - 1} \end{array}} \right| = 0
\Rightarrow \,\, - \alpha + 5\beta + 3\gamma = 0
\Rightarrow \,\,\gamma - 4 + 25\gamma - 10 + 9\gamma = 0
\gamma = \frac{2}{5}
So, \left( {\alpha ,\,\,\beta ,\,\,\gamma } \right) = \left( {\frac{6}{5},\,\,0,\,\,\frac{2}{5}} \right)

80.  (B)

Required area is the area of shaded region (APOQ)


= area of \Delta \,{\text{OAQ}}+ area of sector (OAP)
= \frac{1}{2} \times 4 \times 4\sqrt 3 + \frac{{\pi \left( {4 \times 4} \right)}}{6}
= \left( {\frac{{8\pi }}{3} + 8\sqrt 3 } \right)
81.  (C)
\int {\frac{{{{\sec }^2}x - 2010}}{{{{\sin }^{2010}}x}}dx} = \int {{{\sec }^2}x{{\left( {\sin x} \right)}^{ - 2010}}dx - 2010\int
{\frac{1}{{{{\left( {\sin x} \right)}^{2010}}}}dx} = {I_1} - {I_2}}
Applying by parts on {I_1}, we get
{I_1} = \frac{{\tan x}}{{{{\left( {\sin x} \right)}^{2010}}}} + 2010\int {\frac{{\tan x\cos x}}{{{{\left( {\sin x} \right)}^{2011}}}}dx} +
C
= \frac{{\tan x}}{{{{\left( {\sin x} \right)}^{2010}}}} + 2010\int {\frac{{dx}}{{{{\left( {\sin x} \right)}^{2010}}}} + C}
\Rightarrow {I_1} - {I_2} = \frac{{\tan x}}{{{{\left( {\sin x} \right)}^{2010}}}} + C = \frac{{P\left( x \right)}}{{{{\left( {\sin x}
\right)}^{2010}}}} + C
\Rightarrow P\left( x \right) = \tan \left( x \right)
P\left( {\frac{\pi }{3}} \right) = \tan \left( {\frac{\pi }{3}} \right) = \sqrt 3

82.  (C)
xdy\left( {{y^2}{e^{xy}} + {e^{\frac{x}{y}}}} \right) = ydx\left( {{e^{\frac{x}{y}}} - {y^2}{e^{xy}}} \right)
\Rightarrow \,\,x{y^2}{e^{xy}}dy + {y^3}{e^{xy}}dy = \left( {ydx - xdy} \right){e^{\frac{x}{y}}}
\Rightarrow \,\,{e^{xy}}\left( {xdy + ydx} \right) = \frac{{\left( {ydx - xdy} \right){e^{\frac{x}{y}}}}}{{{y^2}}}
\Rightarrow \,\,{e^{xy}}d\left( {xy} \right) = {e^{\frac{x}{y}}}d\left( {\frac{x}{y}} \right)
Integrate
 {e^{xy}} = {e^{\frac{x}{y}}} + \lambda
\Rightarrow xy = \ln \left( {{e^{\frac{x}{y}}} + \lambda } \right)

83.  (D)
Total ways = {15^7}
For favourable ways, we must select seven coupons from 1 to 9 such that 9 is selected at least once.
Thus total number of favourable ways = number of ways of selecting seven coupons from 1 to 9 – number of ways of
selecting seven coupons from 1 to 8
  = {9^7} - {8^7}
\Rightarrow Required probability = \frac{{{9^7} - {8^7}}}{{{{15}^7}}}

84.  (D)
Here OP = \sqrt {{h^2} + {k^2} + {l^2}} = p
\therefore \,\,{\text{DRs}}\;{\text{of}}\;{\text{OP}}\;{\text{are}}:
\frac{h}{{\sqrt {{h^2} + {k^2} + {l^2}} }},\frac{k}{{\sqrt {{h^2} + {k^2} + {l^2}} }},\frac{l}{{\sqrt {{h^2} + {k^2} + {l^2}} }}
or \frac{h}{p},\frac{k}{p},\frac{l}{p}
Since OP is normal to the plane, therefore, equation of plane is

\frac{h}{p}x + \frac{k}{p}y + \frac{l}{p}z = p\;or\;hx + ky + lz = {p^2}


\therefore \;\;A\left( {\frac{{{p^2}}}{h},0,0} \right),B\left( {0,\frac{{{p^2}}}{k},0} \right),C\left( {0,0,\frac{{{p^2}}}{l}} \right)
Now, Area of \Delta ABC,\;\Delta = \sqrt {A_{xy}^2 + A_{yz}^2 + A_{zx}^2}
Where, {A_{xy}} is area of projection of \Delta ABC on xy plane = area of \Delta AOB
Now, {A_{xy}} = \frac{1}{2}\left| {\begin{array}{*{20}{c}}   {{p^2}/h}&0&1 \\   0&{{p^2}/k}&1 \\   0&0&1 \end{array}} \right| =
\frac{{{p^4}}}{{2|hk|}}
Similarly, {A_{yz}} = \frac{{{p^4}}}{{2|kl|}}\;{\text{and}}\;{A_{zx}} = \frac{{{p^4}}}{{2|lh|}}
\therefore {\Delta ^2} = A_{xy}^2 + A_{yz}^2 + A_{zx}^2,\;\Delta = \frac{{{p^5}}}{{2hkl}}
85.  (B)

f\left( {10 + x} \right) = f\left( {10 - x} \right)


Replace x by 10 – x
f\left( {20 - x} \right) = f\left( x \right)
We have f\left( {20 - x} \right) = - f\left( {20 + x} \right)
\Rightarrow f\left( x \right) = - f\left( {20 + x} \right)
Replace x by 20 + x
f\left( {20 + x} \right) = - f\left( {40 + x} \right)
- f\left( x \right) = - f\left( {40 + x} \right)
f\left( x \right) = f\left( {40 + x} \right)
So f is periodic.
Again
f\left( x \right) = - f\left( {20 + x} \right)
\Rightarrow f\left( { - x} \right) = - f\left( {20 - x} \right) = - f\left( x \right)
Thus f is odd

86.  (A)

Given, y = x\log \left( {\frac{x}{{a + bx}}} \right)


\begin{gathered}    \Rightarrow \frac{y}{x} = \log \left( {\frac{x}{{a + bx}}} \right) \\    \Rightarrow {e^{y/x}} = \frac{x}{{a + bx}}
\\ \end{gathered}
a + bx = x{e^{ - y/x}}
Differentiating w.r.t x
b = x\left( {{e^{ - y/x}}} \right)\left( {\frac{{y - xy'}}{{{x^2}}}} \right) + {e^{ - y/x}}\,\,\,\,\,\,\,\,\,\,\,\,\left( {{\text{where}}\,\,y' =
\frac{{dy}}{{dx}}} \right)
b = {e^{ - y/x}}\left( {\frac{y}{x} - y' + 1} \right)
Again differentiating w.r.t x
0 = {e^{ - y/x}}\left( {\frac{{xy' - y}}{{{x^2}}} - y''} \right) + \left( {\frac{y}{x} - y' + 1} \right)\left( {{e^{ - y/x}}} \right)\left( {\frac{{y
- xy'}}{{{x^2}}}} \right)\,\,\,\,\,\,\left( {{\text{where}}\,\,y'' = \frac{{{d^2}y}}{{d{x^2}}}} \right)
0 = \frac{{y'}}{x} - \frac{y}{{{x^2}}} - y'' + \left( {\frac{y}{x} - y' + 1} \right)\left( {\frac{y}{{{x^2}}} - \frac{{y'}}{x}} \right)
0 = \frac{{y'}}{x} - \frac{y}{{{x^2}}} - y'' + \frac{{{y^2}}}{{{x^3}}} - \frac{{yy'}}{{{x^2}}} - \frac{{yy'}}{{{x^2}}} + \frac{{{{\left( {y'}
\right)}^2}}}{x} + \frac{y}{{{x^2}}} - \frac{{y'}}{x}
0 = - y'' - \frac{{2yy'}}{{{x^2}}} + \frac{{{{\left( {y'} \right)}^2}}}{x} + \frac{{{y^2}}}{{{x^3}}}
0 = - {x^3}y'' - 2xyy' + {\left( {xy'} \right)^2} + {y^2}
{x^3}y'' = {\left( {xy' - y} \right)^2}
or {x^3}\frac{{{d^2}y}}{{d{x^2}}} = {\left( {x\frac{{dy}}{{dx}} - y} \right)^2}
By comparing we get
n = 3\,,\,\,m = 2

87.  (A)

{\text{RHL}}\left( {x = 0} \right) = \mathop {\lim }\limits_{x \to {0^ + }} \,\,\left( {\alpha + \frac{{\sin \left[ x \right]}}{x}} \right) =
\mathop {\lim }\limits_{x \to {0^ + }} \,\,\left( {\alpha + 0} \right) = \alpha
Also, \frac{{\sin x - x}}{{{x^3}}} = \frac{{x - \frac{{{x^3}}}{{3!}} + \frac{{{x^5}}}{{5!}}... - x}}{{{x^3}}} = \frac{{ - 1}}{{3!}} +
\frac{{{x^2}}}{{5!}} - ...
{\text{So,}}\,\,{\text{LHL}}\left( {x = 0} \right) = \mathop {\lim }\limits_{x \to {0^ - }} \,\,\left( {\beta + \left[ {\frac{{\sin x - x}}
{{{x^3}}}} \right]} \right) = \mathop {\lim }\limits_{x \to {0^ - }} \,\,\left( {\beta - 1} \right) = \beta - 1
Since f\left( x \right) is continuous at x = 0\,.
Hence, LHL = RHL
\therefore \,\,\beta - 1 = \alpha
\Rightarrow \beta - \alpha = 1

88.  (D)
For x \in \left( {0,\frac{\pi }{4}} \right)
{x^{2008}}{\left( {\tan x} \right)^{2008}} > {x^{2009}}{\left( {\tan x} \right)^{2009}} > {x^{2010}}{\left( {\tan x} \right)^{2010}}
\therefore \,\,\int\limits_0^{\pi /4} {{x^{2008}}{{\left( {\tan x} \right)}^{2008}}dx} > \int\limits_0^{\pi /4} {{x^{2009}}{{\left(
{\tan x} \right)}^{2009}}dx} > \int\limits_0^{\pi /4} {{x^{2010}}{{\left( {\tan x} \right)}^{2010}}dx}
\Rightarrow {I_1} > {I_2} > {I_3}
\Rightarrow {I_3} < {I_2} < {I_1}
89.  (D)
Given, x = 1 - 3{t^2},\,y = t - 3{t^3}
For, P(−2, 2),
- 2 = 1 - 3{t^2}\,.....\left( {\text{i}} \right)\,\,\,\,\,\,{\text{and}}\,\,\,\,\,\,2 = t - 3{t^3}\,\,\,....\left( {{\text{ii}}} \right)
From (i)
3{t^2} = 3
{t^2} = 1
t = \pm \,1
As t = 1 doesn’t satisfy (ii)
Thus, t = - 1
Now, \frac{{dy}}{{dx}} = \frac{{dy/dt}}{{dx/dt}} = \frac{{1 - 9{t^2}}}{{ - 6t}}
\Rightarrow {\left. {\frac{{dy}}{{dx}}} \right|_{\left( {t = - 1} \right)}} = \frac{{1 - 9}}{6} = - \frac{4}{3}
Thus, equation of tangent at P is
\left( {y - 2} \right) = - \frac{4}{3}\left( {x + 2} \right)
3y - 6 = - 4x - 8
4x + 3y + 2 = 0
Now as this tangent intersects the curve again at Q
Thus,
4\left( {1 - 3{t^2}} \right) + 3\left( {t - 3{t^3}} \right) + 2 = 0
4 - 12{t^2} + 3t - 9{t^3} + 2 = 0
\begin{gathered}    \Rightarrow 9{t^3} + 12{t^2} - 3t - 6 = 0 \\    \Rightarrow \left( {3{t^2} + t - 2} \right)\left( {t + 1} \right) = 0 \\
   \Rightarrow \left( {3t - 2} \right){\left( {t + 1} \right)^2} = 0 \\ \end{gathered}
t = \frac{2}{3} (For Q as t = −1 is for P)
Thus slope of tangent at Q is
\begin{gathered}   {\left. {\frac{{dy}}{{dx}}} \right|_{\left( {t = \frac{2}{3}} \right)}} = \frac{{1 - 9{{\left( {\frac{2}{3}} \right)}^2}}}
{{ - 6\left( {\frac{2}{3}} \right)}} \\    = \frac{{1 - 4}}{{ - 4}} = \frac{3}{4} \\ \end{gathered}
Since, {\left. {\frac{{dy}}{{dx}}} \right|_{\left( {t = - 1} \right)}} \times {\left. {\,\,\frac{{dy}}{{dx}}} \right|_{\left( {t = \frac{2}{3}}
\right)}} = \frac{{ - 4}}{3} \times \frac{3}{4} = - 1
Hence, tangents at P and Q are at right angle.

90.  (B)

\frac{{dx}}{{dt}} = \frac{{{l_1}}}{{10}}
Now, \tan \theta = \frac{{{l_2}}}{x} = \frac{y}{{{l_1}}}
\Rightarrow \,\,y = \frac{{{l_1}{l_2}}}{x}
\Rightarrow \,\,\frac{{dy}}{{dt}} = - \frac{{{l_1}{l_2}}}{{{x^2}}} \cdot \frac{{dx}}{{dt}}
\Rightarrow \,\,{\left. {\frac{{dy}}{{dt}}} \right|_{x\, = \,\frac{{{l_1}}}{2}}} = \frac{{ - 4\,{l_1}{l_2}}}{{l_1^2}}\left( {\frac{{{l_1}}}
{{10}}} \right) = - \frac{{2{l_2}}}{5}m/sec

You might also like